ChaseDream

标题: 这题的答案也太不像了把~ [打印本页]

作者: angelashao12    时间: 2006-8-16 02:10
标题: 这题的答案也太不像了把~

GWD 23 39

In Gandania, where the government has a monopoly on tobacco sales, the incidence of smoking-related health problems has risen steadily for the last twenty years.  The health secretary recently proposed a series of laws aimed at curtailing tobacco use in Gandania.  Profits from tobacco sales, however, account for ten percent of Gandania’s annual revenues.  Therefore, Gandania cannot afford to institute the proposed laws.

 

Which of the following, if true, most seriously weakens the argument?

 

  1. All health care in Gandania is government-funded.

  2. Implementing the proposed laws is not likely to cause a significant increase in the amount of tobacco Gandania exports.

  3. The percentage of revenue Gandania receives from tobacco sales has remained steady in recent years.

  4. Profits from tobacco sales far surpass any other single source of revenue for the Gandanian government.

  5. No government official in Gandania has ever previously proposed laws aimed at curtailing tobacco use.

答案居然是A

我选B

这题错了把~

大家看看把

~


作者: lawyer_1    时间: 2006-8-16 05:47

理解改题的答案为A的关键是理解如何算WEAKEN.WEAKEN是使原文依据原文的证据所得出的结论的说服力被削弱.改题如果A是真的话,则政府要为吸烟引起的健康问题买单,禁烟一方面使收入减少,另一方面使健康支出减少,到底哪个减少的多就难说啦,所以结论的说服力就没那么强啦,所以削弱.

B无关,因为原文没有给出烟草的出口量的财政收入所占该政府收入的情况,所以无关


作者: eejensen    时间: 2006-8-16 17:07
of course A

作者: angelashao12    时间: 2006-10-15 16:14

谢谢LAWYER大牛牛~~

 






欢迎光临 ChaseDream (https://forum.chasedream.com/) Powered by Discuz! X3.3